Question on observer created reality

In summary: Of course, this means that there is no objective reality, just as there is no objective knowledge. In summary, the Copenhagen view of quantum theory is that there is no real world, just observation.
  • #106
good thread ;)

gptejms

Did you answer Zappers question or did I miss that? I was about to ask the same thing. What is the nature of this potential? - That is introduced. There are only 2 things you could say, I want to here you say what.

vanesch #94

The QM - Classical boundary solution lies with the wavefunction(s) amplitude of a given construct. Mass is explicitly related to this function, such that the less mass an "object" has the greater the potential wavelength attributes and thus the greater the probability of P and M. Once you reach a certain density of a collection of wavefunctions in a given construct - more mass - the "object" ceases to have a QM wavefunction when considering in context of a whole object. It is at the size of molecules and compound atoms that the wavefunction amplitude of the whole decreases to a point where the molecules probability of location is almost certain.

Ask yourself, could a neutron for example, qualify for wavelengths attainable by photons based on its mass alone? I think not, and the reason being because its mass does not allow it such degree of freedom. It has as it is, a well defined position because of its larger mass and any change in momentum is bounded by such mass.

The question is, a what point when mass equals near zero does the wavelength have almost unlimited wavelength potential? The photon of course, radio waves are extremely large and qualify as one of the least massive objects we know of. Going up several notches to gamma rays we see that its wavelength amplitude is far more focused - a more well defined position, reason being because it has more mass but instead in the form of "energy" as found in Einstiens famous mass energy equations.

Now that I've said that, when we talk about measurements on massive objects we know that it has Already a well defined almost absolute position, not quite so in the QM world as things evolve more rapidly then we are able to observe them. There is no such thing as a collapse of the wavefunction in the classical macro world, because the wavefunction is already collapsed (reduced) to a point where it effectivly has none. Interference can't be considered because interference requires the interference of wavefunctions of individual particles, when an object doesn't Have a wavefunction ... you get the idea. Covalent bonds are the process by which atoms collectively combined their individual normal mode wave resounaces to form a new wavefunction as a compound atom, a superposition perhaps but not in the normal sense.
 
Physics news on Phys.org
  • #107
vanesch said:
It surely is very difficult if you want the full treatment in QFT, for many reasons. The physical picture behind it is the the E-fields in the right transition mode(s) in QFT are non-zero even in vacuum (the expectation values of the squares of the amplitudes of these modes are not zero), and it is the interaction of the electon of the atom with these modes that makes that the stationary states of the atom when calculated, taking purely into account the coulomb interaction with the nucleus, are NOT stationary states anymore when taking into account the dynamical EM field (as a quantum field).

Not QFT,I wanted to see it in the Heisenberg picture of non-rel. QM.Anyway, the points you have raised are good.

The time evolution will now do the following thing:

at t = t2 > t0, we have that the wave function equals:

psi(x) = a H0(x) exp(-i E0 t2) + b H1(x) exp(- i E1 t2) + c H2(x) exp(- i E2 t2) ...

This will of course now not take on the aspect any more of a plane wave.

All this you've said earlier.My question was this:-the x-part of the plane wave is exp(ikx)-you decompose it into a sum of Hermite Gauss functions--this sum remains as such no matter how much you evolve it--so in your picture the momentum i.e. hbar k remains conserved--should it be?
Another thing:-the t part is exp(-i omega t)--you can't decompose a single frequency plane wave in terms of plane waves of other frequencies.
 
  • #108
vanesch said:
I'll try to think up a simple "analogy".
Consider the free non-relativistic hamiltonian of 2 particles. We could think of an equivalent of "relativity" that forbids us to have dynamical interaction between the two particles (instead of between spacelike separated parts of quantum fields). This would be formalized by imposing commutation between all observables that relate to particle 1 and all observables that relate to particle 2 in the Heisenberg picture. So as long as we work with product states (in the Schroedinger picture) we remain in product states.
But this doesn't stop us from considering entangled states ! And then suddenly we do find EPR like correlations between both. Is this incompatible with our imposed absence of dynamical interactions between both ?
No, it is just a property of the quantum theory.
In the same way, the absence of dynamics between different, spacelike separated parts of a quantum field do not forbid you to have them entangled, and as such find EPR correlations between them.

That's right.
Let me add that one has to give a mechanism/reason for the entanglement, which exists at the start(in an EPR experiment).And since it's just the polarization that's being measured,there's no violation of microscopic causality anyway(see my earlier post).
 
  • #109
gptejms said:
All this you've said earlier.My question was this:-the x-part of the plane wave is exp(ikx)-you decompose it into a sum of Hermite Gauss functions--this sum remains as such no matter how much you evolve it--so in your picture the momentum i.e. hbar k remains conserved--should it be?

Before asking to the others, you should pay attention to what is written since the beginning (i.e. make an effort). If you do not want to accept/understand the mathematical results of more than 100 years of developpment in QM it will be difficult for you to make valid claims/interpretations.

Look at what Patrick has written in its post #104. This is a basic QM application. For example think on what the energy of a free particle is and what the energy of particle with a potential that depends on time is as well as the cost to have a pure plane wave at t=0 (for example think with the Heisenberg view).

Seratend.
 
  • #110
seratend said:
Before asking to the others, you should pay attention to what is written since the beginning (i.e. make an effort). If you do not want to accept/understand the mathematical results of more than 100 years of developpment in QM it will be difficult for you to make valid claims/interpretations.

I think you are the one who needs to pay attention to what is written--have you understood what's being discussed or what I asked vanesch--make an effort!If you do that you'll understand that no mathematical result of more than 100 years is being challenged.
 
  • #111
gptejms said:
I think you are the one who needs to pay attention to what is written--have you understood what's being discussed or what I asked vanesch--make an effort!.

I make an effort. Reread and try to understand post #104 by yourself. If you make an effort to understand what is written, your question about the momentum should be answered.

Seratend.
 
  • #112
Perhaps my using the word plane wave in the following has caused confusion.In post #107 I wrote:-
Another thing:-the t part is exp(-i omega t)--you can't decompose a single frequency plane wave in terms of plane waves of other frequencies.

Read this as:-
Another thing:-the t part is exp(-i omega t)--you can't decompose a single frequency sinusoid in terms of sinusoids of other frequencies.

Right,seratend?
 
  • #113
tdunc said:
Once you reach a certain density of a collection of wavefunctions in a given construct - more mass - the "object" ceases to have a QM wavefunction when considering in context of a whole object.

That is not standard QM. It may very well be right, but you have a whole lot of stuff to explain.

Ask yourself, could a neutron for example, qualify for wavelengths attainable by photons based on its mass alone? I think not, and the reason being because its mass does not allow it such degree of freedom. It has as it is, a well defined position because of its larger mass and any change in momentum is bounded by such mass.

Well, here's for example already a difficulty in exactly the thing you cite. Just at the other side of my office, there's a small angle neutron scattering machine, which produces interference patterns in neutron scattering due to density correlations over several micrometers. So clearly the neutron behaves as a quantum object at least over these distances.
We have quantum states of neutrons in a gravitational potential (ultracold neutrons), which extend over several tens of micrometers too.

And then there are of course collective quantum phenomena, such as superfluids, which have macroscopic masses.

It's not so easy to define a classical-quantum boundary!

cheers,
Patrick.
 
  • #114
tdunc said:
good thread ;)

gptejms

Did you answer Zappers question or did I miss that? I was about to ask the same thing. What is the nature of this potential? - That is introduced. There are only 2 things you could say, I want to here you say what.

The potential will depend on what measuring device you are using,what is the nature of the interaction etc.I don't want to introduce an ad-hoc potential and do a calculation--any potential would alter the wavefunction(and disturb the interference pattern)-we need a potential that shrinks the wavefunction to span one slit only and the potential should be realistic.The case I was considering was Heisenberg's microscope at one of the slits--so photons are interacting with the electron.The crudest way to formulate this could be to take the potential to introduce random kicks in time i.e. be a sum of delta functions in time(Sum_i V_i delta(t-t_i) where t_i 's are randomly distributed--may be V_i 's could be given some spatial dependence(?).Frankly I don't have a good model for this.

It's better to take recourse to QFT so that you are in a fully quantum scenario.A consequence of this is discussed in my earlier post titled 'QFT and the measurement problem'.Briefly--when you make a measurement say at one slit, you create a disturbance that travels at the velocity of light to other places and the field readjusts to new values corresponding to what is observed-no interference pattern at the screen.
 
Last edited:
  • #115
ok vanesch I've just read what you wrote in post #104--I withdraw my question.
 
  • #116
vanesch

Standard QM is not something that I am known for ;) I have a whole lot to explain do I? Sounds familiar..

Your examples did not contradicted my concept.


gptejms # 114

Well I was more concerned When and Where the "measurement" as you define it takes place, exactly before or after or at the 2-slit, knowingly or not. Or in another case you could have considered, as some do, the 2-slit to Be the measurement and measurement tool. Clearly though if you were to subject the particle to a measurement - a measuring device, you could do this at Any point, the results will be quite different mind you, that's why I wanted to clairify, but it all depends what concept of behavior you are trying to convey that we don't already know? Or what theory are you promoting, there's plenty of them.

"Briefly--when you make a measurement say at one slit, you create a disturbance that travels at the velocity of light to other places and the field readjusts to new values corresponding to what is observed-no interference pattern at the screen."

Right so? You are trying to convey that the collapse of the wavefunction is at the speed of light correct?

What if I said though that the speed of a photon moving forward - c, is not the same as the speed at which a photon oscillates? Because that is afterall the varible that will collapse, not the forward speed varible. Oh it Could be, but what says that it is? I can imagine that if I throw a baseball that wobbles slightly up and down as it goes, is the speed of the wobble (call it its wavefunction) the speed of forward propagation? no

What I might say though _for the time being_ is that the collapse of a photons wavefunction is Limited by the speed of light so that we can avoid any what if senerios that deal with very large spaces and superluminal collapse.
 
  • #117
gptejms said:
That's right.
Let me add that one has to give a mechanism/reason for the entanglement, which exists at the start(in an EPR experiment).

That mechanism (to obtain the entangled state from the start) is obtained by dynamical interaction when the two subsystems WERE interacting. Interaction (hamiltonian interaction) causes entanglement of the system.
When the subsystems get separated, they cannot interact dynamically anymore, but their global state is still entangled.
The interaction with a measurement apparatus can be seen exactly in the same way (it is the von Neumann "pre-measurement interaction").

cheers,
Patrick.
 
  • #118
tdunc said:
gptejms # 114

Or what theory are you promoting, there's plenty of them.

I am not promoting any theory here.I can tell you what my train of thought has been during this thread:-

1.Measurement is akin to introducing a potential which shrinks the wavefunction to one or the other slit--it's definitely 'one slit'(no interference pattern) now,my conscious knowledge of it is not important.It's just that when I go to observe,it's revealed to me that the particle went through this particular slit--I may go after 10 years to make an observation that there is no interference pattern and that the particle went through this particular slit.This does not mean that the particle was in a superposition of two slits all this while.Now one may ask how the potential shrinks the wavefunction sometimes to slit 1 and at other times to slit 2--well the potential has an element of randomness in it.The merit of this scheme is that measurement is explained via QM/unitary evolution itself.

2.Then to be in a fully quantum scenario,I thought why not look at this problem from the QFT aspect.There, from the principle of microscopic causality,it becomes clear that measurement at one place introduces a disturbance which causes the field at other places to readjust to new values within a time dictated by the velocity of light.This at least tells you that the so called collapse can't be at superluminal speeds.EPR is not in contradiction with this because you are not measuring the field,you are just measuring the polarizations which are correlated due to the entanglement.But one case stands out:-say you have a single particle in a box,you put a partition in the box,take one half to Pluto,keep the other half on earth,make a measurement on Earth and find the particle.Now microscopic causality tells me that this information is not instantaneously passed on to Pluto--the field there will readjust to 'no particle' after some time gap.But if an observer on pluto makes a measurement before that,can he still find the particle?--I leave that unanswered.

3.Does the above i.e. microscopic causality solve the measurement problem.No,it doesen't.Now the wavefunction is a functional of the field.There is still a chance for the particle to be in this or that slit!Now I do not know enough QFT to introduce another field to interact with the particle field and to see what that leads to.I hope the QFTists here can shed some light on that.
 
  • #119
vanesch said:
That mechanism (to obtain the entangled state from the start) is obtained by dynamical interaction when the two subsystems WERE interacting. Interaction (hamiltonian interaction) causes entanglement of the system.
When the subsystems get separated, they cannot interact dynamically anymore, but their global state is still entangled.

Agree with this part.
 
  • #120
Continuing with what I wrote to tdunc,here's point 4

4. In point no. 1,I said that the potential has an element of randomness in it.Now coming to QFT,the quantum field by its very nature is stochastic--the result of a measurement at one place has a probability attached with it.So let's use that to our advanatge--when I make a measurement at one of the slits,I sometimes find the electron and sometimes don't(in which case it's at the other slit)--this is exactly what the stochastic potential(in point no. 1) was supposed to do!

So I think a satisfactory picture is this:-if you don't make any measurement what-so-ever,it's better to think just in field terms--the two slits cause a field readjustment at the screen to form the interference pattern.When you make a measurement at one of the slits,the field is again disturbed and readjusts to 'no electron' at the other slit and 'no interference effect' on the screen.

It's just field all the way,only measurements/interactions are quantum.Appearance of the interference pattern by introduction of the two slits,the disturbance/destruction of this pattern by introducing the measurement device at one of the slits is just 'field readjustments' that proceed at the speed of light.I think it's not even justified to say that the electron passed through both slits--you can talk of 'an electron' only when you make a measurement at one of the slits and when you do that,there is no interference pattern.Remember complementarity--it's a perfectly sound principle.Think of the whole thing as just field readjustments(measurement or no measurement) and it's even better.

What do you all say to the above?
 
Last edited:
  • #121
gptejms

"Now one may ask how the potential shrinks the wavefunction sometimes to slit 1 and at other times to slit 2--well the potential has an element of randomness in it."

How does the 'potential' shrink the wavefunction period. I could care less what slit it goes through and the fact that its random, I already know the mechanics of that.

Ok 2-slit experiment aside. We know that in the case of a "detection area" manifested by a measuring device the wavelength of a particle(s) is reduced to no more than the width of the detection area. If the displacement of the particle is already <= the width of the detection area, is does not collapse. If it is greater than, it collapses. Supposedly. Why and How? This was a lingering question I had not answered myself in the past.


found it

"A recent paper by Keller (6) demonstrated rigorously for a propagating particle that its wave function, as an amplitude for location in space, is collapsed by any detection. Using probability theory, he proved that the probability amplitude wave for location of the ongoing quantum particle immediately following the observation becomes limited to the observation area, in other words its effective size is collapsed to that of the observation area."

"AJP 1990 Joseph B. Keller "Collapse of wavefunctions and probability densities"

I can't read it because I don't have a subscription.
 
  • #122
Radio antennas > collapse the wavefunction > to the area of detection - the antenna itself > limited by the speed of light. Do you have an explanation for this?
 
  • #123
Patrick,would you like to comment on my post #120?
 
  • #124
tdunc said:
How does the 'potential' shrink the wavefunction period. I could care less what slit it goes through and the fact that its random, I already know the mechanics of that.

There are 4 points there,go through all of them and then comment.I need to see some comments on point 4,which is the most important.

"A recent paper by Keller (6) demonstrated rigorously for a propagating particle that its wave function, as an amplitude for location in space, is collapsed by any detection. Using probability theory, he proved that the probability amplitude wave for location of the ongoing quantum particle immediately following the observation becomes limited to the observation area, in other words its effective size is collapsed to that of the observation area."

"AJP 1990 Joseph B. Keller "Collapse of wavefunctions and probability densities"

I can't read it because I don't have a subscription.

I don't have a subscription either.
 
  • #125
gptejms said:
So I think a satisfactory picture is this:-if you don't make any measurement what-so-ever,it's better to think just in field terms--the two slits cause a field readjustment at the screen to form the interference pattern.When you make a measurement at one of the slits,the field is again disturbed and readjusts to 'no electron' at the other slit and 'no interference effect' on the screen.

It's just field all the way,only measurements/interactions are quantum.Appearance of the interference pattern by introduction of the two slits,the disturbance/destruction of this pattern by introducing the measurement device at one of the slits is just 'field readjustments' that proceed at the speed of light.I think it's not even justified to say that the electron passed through both slits--you can talk of 'an electron' only when you make a measurement at one of the slits and when you do that,there is no interference pattern.Remember complementarity--it's a perfectly sound principle.Think of the whole thing as just field readjustments(measurement or no measurement) and it's even better.

What do you all say to the above?

This is a "local realist" approach, and some people try to work it out ; for instance, the promotors of "stochastic electrodynamics" and so on. It is considered a bit "fringe" work (but that by itself doesn't matter).
The big difficulty is to make *explicit* models of what's going on, in order to obtain the same results as standard QM.
And the really nasty part are EPR situations (that's often why these people simply deny these experiments and try to find loopholes in it). They imply then always faster-than-light effects in whatever is the explicit "collapse" mechanism.
Mind you, I don't find this approach totally ridiculous - in fact my hope is that gravity does exactly this, on some level or other. But this would in any case mean a serious deviation from as well general relativity as from quantum theory as we know it.

One thing is clear, however: this approach is not completely compatible with quantum theory "all the way up".
 
  • #126
vanesch said:
This is a "local realist" approach, and some people try to work it out ; for instance, the promotors of "stochastic electrodynamics" and so on. It is considered a bit "fringe" work (but that by itself doesn't matter).

The kind of local realism I am talking of is dictated by the principle of microscopic causality(i.e.nothing but the commutation property of the field at two space-time points),which is the basic feature of QFT--how can anyone doubt it?And this kind of local realism dictated by microscopic causality does not contradict EPR situations as I have already said--in EPR you don't measure the field anywhere,you are just measuring polarization at two places (which due to entanglement leads to higher correlation than expected on classical grounds).


Mind you, I don't find this approach totally ridiculous - in fact my hope is that gravity does exactly this, on some level or other. But this would in any case mean a serious deviation from as well general relativity as from quantum theory as we know it.

Don't know what you are saying here.
 
  • #127
gptejms said:
The kind of local realism I am talking of is dictated by the principle of microscopic causality(i.e.nothing but the commutation property of the field at two space-time points),which is the basic feature of QFT--how can anyone doubt it?And this kind of local realism dictated by microscopic causality does not contradict EPR situations as I have already said--in EPR you don't measure the field anywhere,you are just measuring polarization at two places (which due to entanglement leads to higher correlation than expected on classical grounds).

The point is: if you try to obtain the "wave function collapse" by a dynamical process which obeys what you call microscopic causality, you should see that you then have a serious problem with EPR situations. I don't understand your remark: "are just measuring polarisation at two places, you don't measure the field" ? What else is polarization but an aspect of the field ??
But again, in QFT, the microscopic causality puts limits on the DYNAMICAL INTERACTIONS that can be present in the Hamiltonian ; they do not specify anything about what happens to the wave function, of which the collapse (if this collapse is considered real) is global and does not obey any microscopic causality. So I don't see how you are going to obtain the last one effectively based upon the first one.
The only way out (IMHO) is by keeping unitary evolution "all the way up" if you stick to microscopic causality, and to have an other explanation for the *apparent* collapse - at least if you want to stick some meaning to the wavefunction in the first place (which you don't have to in a purely epistemological view).
I really don't see how you are going to do the collapse thing unitarily (with a hamiltonian that obeys local causality). EPR for me seems to be a killer of that idea (although it is not the only one).

But I think it is even worse. As long as you accept unitary evolution, I don't see how you can get rid of the linearity that a |1> + b|2> will give you a result that is a |result_of_state_1> + b |result_of_state_2>.
And if |result_of_state_1> includes having the pointer of your voltmeter on 11V and |result_of_state_2> is having the pointer of your voltmeter on 23V, I don't see how you're going to get away with having sometimes 11 V and sometimes 23 V.

cheers,
Patrick.
 
  • #128
vanesch said:
The point is: if you try to obtain the "wave function collapse" by a dynamical process which obeys what you call microscopic causality, you should see that you then have a serious problem with EPR situations. I don't understand your remark: "are just measuring polarisation at two places, you don't measure the field" ? What else is polarization but an aspect of the field ??

There is a difference:-microscopic causality implies that 'field measurements'(field intensity)' at space-like intervals can't be correlated.An aspect of the field like polarization can be--polarizations can be entangled,the field intensities themselves can't be entangled for space-like intervals.

But again, in QFT, the microscopic causality puts limits on the DYNAMICAL INTERACTIONS that can be present in the Hamiltonian ; they do not specify anything about what happens to the wave function, of which the collapse (if this collapse is considered real) is global and does not obey any microscopic causality. So I don't see how you are going to obtain the last one effectively based upon the first one.
The only way out (IMHO) is by keeping unitary evolution "all the way up" if you stick to microscopic causality, and to have an other explanation for the *apparent* collapse - at least if you want to stick some meaning to the wavefunction in the first place (which you don't have to in a purely epistemological view).

Consider the following:-

There is a source of electrons and a screen a little away.Think of this as just a field,an 'electron field'.There is no pattern on the screen.You introduce the double slit in between the source and the the screen.You have imposed certain constraints,the field gets disturbed and readjusts to 'interference pattern' on the screen--this proceeds at the speed of light(consistent with microscopic causality).Till I do not make any measurement,I think of it as just a field--in fact I am not even allowed to think of it as particles,until I make a measurement and find a particle.In fact my assertion is that only interactions are quantum,otherwise it's just a field.When I make a measurement at one of the slits and find a particle,the field again is disturbed and readjusts to 'no particle' at the other slit(if one particle comes at a time) and 'no interference effects' on the screen.Now that I have found the particle,I am allowed to talk in terms of the particle picture--and I don't see any contradictions--there is no interference pattern on the screen.

The problem arises only when you think of it as a particle incident on the double slit--you write down a|1> + b|2> and think how this collapses by unitary evolution.I say that there's no particle until you make a measurement,till then it's all a field.It's as if the measurement 'creates the particle',before that it doesen't even exist.

Jagmeet Singh
 
  • #129
Apak said:
Before I begin I feel that I should state that I have only a passing amount of knowledge on the subject so if I've got something wrong let me know. My current understanding of QM is that certain atomic attributes are created through observation of these attributes. This seems to suggest that there are two values present in an observation, the observer and the reality created. So my question is what special properties does the observer possesses that allows an observation to be made. Here's what I mean, when a reality is present an observer is necessarily present. However, if an observer is present it does not neccesitate the presence of a reality. Looked at from this perspective the observer can be considered real, in that regardless of observations being made the observer always exist's and that reality only exists in relation to the observer. This would mean that the constituent parts of the observer in no way find their primary causes in the reality which the observer creates. This means that the system the observer creates finds all its values in the observer. The observer on the other hand finds none of its initial values in the system it creates. This is what troubles me the most about QM, if there is no separation between observer and observed, and the observer finds its initial values bound up within a system it creates through observation how is anything ever observed? Please help.

I don't have any amount on the subject which probably explains why I don't have any formal training. Which probably explains why I don't understand 90% of the replies, but when I read your post, these thoughts came to minf.

1)
Apak said:
This seems to suggest that there are two values present in an observation, the observer and the reality created."

A) I don't understand QM but I don't think that reality gets "created". When you mention 2 values, the first thing that comes to my mind is the space-time co-ordinate of the observer and the s-t co-ordinate of that which is being observed. The reason this came to mind is that the only way you can observe anything, or as you say, create reality is when a photon traverses s-t to go from that which is being observed directly into the retinae of the observer. But since each photon that is absorbed into the retina of an observer at a specific s-t co-ordinate is unique then I would think that "reality" as we define it is a one-of-a-kind but slightly inaccurate snapshot of an object's past state at a precise s-t co-oridinate. My logic for this definition is based on the following:

1) Its a past representation because as it traverses s it must also traverse t

2) Its inaccurate because

a) prior to leaving the object the photon imparts a recoiling force upon it which must displace it and hence change its state by an undeterminate factor.

b) The photon gets tugged upon by gravity and who knows what else as it traverses s-t to get to your retina.

3) Its one of a kind because

a) you can never have 2 photons strike an object exactly in the same spot, at the same velocity, same approach angle, same gravitational lensing, etc...

a) Even if you could have 2 exactly the same photons You will never get 2 brains that will extrapolate the same representation.

In this context, reality is relative to the observer, which is to say that essence precedes existence. I think therefore I am.

2)
Apak said:
what special properties does the observer possesses that allows an observation to be made. Here's what I mean, when a reality is present an observer is necessarily present. However, if an observer is present it does not neccesitate the presence of a reality

A) By properties I would say a pair of eyes connected to a brain that can take take in photons and extrapolate.

3)
Apak said:
Here's what I mean, when a reality is present an observer is necessarily present.

A) As long as by reality you mean something that we observe. But even if it is not observed it must still exist. I understand that quantum mechanics says something else and so do a lot of others including Descartes, to whom I would counter: Nay Rene! Sum ergo cogito! And then maybe slap him with my glove for effect...

4)
Apak said:
if an observer is present it does not neccesitate the presence of a reality

A) Again I would counter that simply because the only place I would think where reality is not present is anything outside the boundaries of our universe as defined by s-t. So anything that does not have an s-t co-ordinate cannot be considered "real". This does not include any s-t region that might be causally disconnected from us due to the fact that it is receding faster than the speed of light. Imo reality does not require c to give it essence or an s-t co-ordinate even though a photon from that co-ordinate will never reach the earth. Reality simply is. Which is really just a way of saying that then s is dependent on t but independent of v...which it must be because as soon as you get v involved then we're talking about that one-of-a-kind but slightly inaccurate space-time representation of reality and not the real reality (the real reality...I made a funny!) Another argument I would make is that if an observer is present, then the observer themselves define reality. Again, this is a case where existence precedes the essence of reality and not vice-versa (i.e. sum ergo cogito)

5)
Apak said:
This means that the system the observer creates finds all its values in the observer. The observer on the other hand finds none of its initial values in the system it creates. This is what troubles me the most about QM, if there is no separation between observer and observed

A) Same argument. I can see why you are having trouble with it. I'm having a lot of trouble with it myself. Maybe I'm just not getting it (likely) but it seems to me like this is a case of human arrogance (unlikely because these humans are like 1000 times smarter th an me) to try to define reality through us. In this sense it almost seems like thir is a prescriptive approach rather than a descriptive one. I prefer the latter. But I have a sneaking suspicion that i am being fooled by my limited concept of reality and also not understanding what it is that QM is really saying. This is likely because I know that QM does not suggest that there is no separation between the observer and the observed because such a position goes against what that ice-berg lettuce guy said. (sorry sometimes I recall the mnemonic and not the memory which is like saying...that the...nevermind...)
 
Last edited:
  • #130
In continuation of what I wrote in post #128,let me add the following:-

In QFT,what is the role of the wavefunction?Now, wavefunction is a functional of the field--what this just does is to give you the probability density of finding a certain field value(upon measurement) at a place.You can't have a superposition of this wavefunction at two places--we don't know what meaning to assign to it.We no longer have the wavefunction of a particle which leads to the measurement problem.

Now let me come to the Schrodinger equation which gave rise to the notion of a wavefunction(of a particle).Schrodinger equation is the non-relativistic limit of the K.G. equation or the Dirac equation which are field equations.Now does the field suddenly change into a wavefunction when the velocity is sufficiently lowered i.e. v/c < < 1? Of course not,it's still a field which obeys the continuity equation.It also satisfies microscopic causality and the wavefunction is still a functional of the field.Wavefunction of a particle is a notion that leads to all the problems--it should be replaced by 'field of a particle' or particle field.

Jagmeet
 
Last edited:
  • #131
gptejms said:
In QFT,what is the role of the wavefunction?Now, wavefunction is a functional of the field--what this just does is to give you the probability density of finding a certain field value(upon measurement) at a place.You can't have a superposition of this wavefunction at two places--we don't know what meaning to assign to it.We no longer have the wavefunction of a particle which leads to the measurement problem.

No, the wavefunction is indeed a functional of the field (in the rarely used Schroedinger picture of QFT), but it gives you the probability AMPLITUDE to have a CERTAIN ENTIRE CLASSICAL FIELD CONFIGURATION.
So you can have a superposition of the kind:

a |plane wave with wavevector k> + b |sphericalwave outward from A> + ...

You can have a superposition of different fields, for instance a field that has a bump at A, and another field that has a bump at B. This is not the same as a field that has both a bump at A and a bump at B.
Now, when a measurement establishes (in the first case) that we have a bump at A, then the collapse comes down to the component in the wave function (field with the bump at B) disappearing from the wavefunction, instantaneously. This is exactly the same situation as in non-relativistic QM, so I don't see how this "resolves" the measurement problem.

Now let me come to the Schrodinger equation which gave rise to the notion of a wavefunction(of a particle).Schrodinger equation is the non-relativistic limit of the K.G. equation or the Dirac equation which are field equations.Now does the field suddenly change into a wavefunction when the velocity is sufficiently lowered i.e. v/c < < 1? Of course not,it's still a field which obeys the continuity equation.

This view (Dirac's view) is known to be misguided now - although it took until the 50ies for people to fully realize this. The Schroedinger equation is not the NR limit of the KG equation or the Dirac equation in QFT. The KG or the Dirac equation play the role of Newton's equations of motion and the Schroedinger equation remains what it is: the dynamical prescription of the wavefunction in Hilbert space (which is now constructed over field states instead of particle positions).

It also satisfies microscopic causality and the wavefunction is still a functional of the field.Wavefunction of a particle is a notion that leads to all the problems--it should be replaced by 'field of a particle' or particle field.

"Field of a particle" in QFT is the CLASSICAL FIELD. And on this classical field, one applies now "the wavefunction" over all possible classical configurations of the field. The Dirac equation is NOT the relativistic version of the Schroedinger equation, it is the classical dynamical equation of the classical field that has to be quantized. It is the CLASSICAL field evolution equation (Dirac's equation) that satisfies microscopic causality, but this classical field is NOT the wavefunction.

cheers,
Patrick.
 
  • #132
vanesch said:
No, the wavefunction is indeed a functional of the field (in the rarely used Schroedinger picture of QFT), but it gives you the probability AMPLITUDE to have a CERTAIN ENTIRE CLASSICAL FIELD CONFIGURATION.

Right!Thanks,I stand corrected.

You can have a superposition of different fields, for instance a field that has a bump at A, and another field that has a bump at B. This is not the same as a field that has both a bump at A and a bump at B.

Why 'different fields'--you can have a superposition of two different field configurations,where the field is the same.

Now, when a measurement establishes (in the first case) that we have a bump at A, then the collapse comes down to the component in the wave function (field with the bump at B) disappearing from the wavefunction, instantaneously. This is exactly the same situation as in non-relativistic QM, so I don't see how this "resolves" the measurement problem.

Seems fine.But the difficulty is that this seems to violate microscopic causality(a property of any quantum field) namely that if you introduce a measurement/disturbance at A,the field at B can not get instantaneously readjusted. Before a measurement is made,the field at B could have been a bump or no bump,now it definitely is 'no bump' in zero time.


This view (Dirac's view) is known to be misguided now - although it took until the 50ies for people to fully realize this. The Schroedinger equation is not the NR limit of the KG equation or the Dirac equation in QFT. The KG or the Dirac equation play the role of Newton's equations of motion and the Schroedinger equation remains what it is: the dynamical prescription of the wavefunction in Hilbert space (which is now constructed over field states instead of particle positions).

See what I am asserting here is that the Schrodinger equation is the NR limit of the KG or Dirac field(quantum) equations--where the field is quantised.There is nothing like the wavefunction of a particle.Plus the field obeys microscopic causality.


It is the CLASSICAL field evolution equation (Dirac's equation) that satisfies microscopic causality, but this classical field is NOT the wavefunction.

What do you mean here?It's not the classical field,but the quantum field that obeys microscopic causality(commutation relations of the field).
 
  • #133
gptejms said:
The problem arises only when you think of it as a particle incident on the double slit--you write down a|1> + b|2> and think how this collapses by unitary evolution.I say that there's no particle until you make a measurement,till then it's all a field.It's as if the measurement 'creates the particle',before that it doesen't even exist.

That's how I think of it. The measurement itself is the particle. It might
be reasonable in some situations to speak of particles or waves or
whatever existing independent of measurement, but, strictly speaking,
these terms refer to recorded instrumental phenomena (and values
in formalizations associated with instrumental preparations).

Precisely what might exist, and how it might behave, in some
'quantum realm' beyond or independent of macroscopic records
is an open question. With both slits open, there's currently
no way to tell whether an assumed quantum disturbance incident
on the double-slit went through both slits or only one.

In an unambiguous usage of terms, it isn't just "as if" the photon
or electron doesn't exist until a recorded measurement creates it -- it
really *doesn't exist* in the world (beyond mathematical form) of
physical interactions until it becomes physical evidence via some
instrumental change(s).

I'm thinking of the measurement problem as a real, physical
incapability on our part. Your proposed resolution to this problem
seems to me to be a metaphysical, and not satisfactory, one.
 
Last edited:
  • #134
vanesch said:
Now, when a measurement establishes (in the first case) that we have a bump at A, then the collapse comes down to the component in the wave function (field with the bump at B) disappearing from the wavefunction, instantaneously. This is exactly the same situation as in non-relativistic QM, so I don't see how this "resolves" the measurement problem.

Let me add here the following to what I already wrote in a part of post #132 as a response to the above.You are writing your wavefunction as a superposition of a 'field bump' at x1,t1 and a 'field bump' at x2,t1(i.e. two possible field configurations).Now your assertion is that when I make a measurement at x1,t1 and the bump appears there,the bump at x2,t1 is insatantaneously wiped off.

My assertion is that if at all you have to use your wavefunction for field superpositions,you can't use equal times at x1 and x2.At x2,the time must at least be (x2-x1)/c + t1 (in consistence with microscopic causality).

Let me repeat that the Schrodinger equation is an equation for a quantum field not for wavefunction.Wavefunction in the new picture(if I may say so) has a lesser role to play--you can't use it for superpositions in the way one did for the wavefunction of a particle.
 
Last edited:
  • #135
Sherlock said:
In an unambiguous usage of terms, it isn't just "as if" the photon
or electron doesn't exist until a recorded measurement creates it -- it
really *doesn't exist* in the world (beyond mathematical form) of
physical interactions until it becomes physical evidence via some
instrumental change(s).

I'm thinking of the measurement problem as a real, physical
incapability on our part. Your proposed resolution to this problem
seems to me to be a metaphysical, and not satisfactory, one.

Would it be ok for you if I dropped the "as if" ?!Is that the only objection?Keeping it or dropping it is just a matter of taste--stick to whatever you like,it makes no difference.
 
  • #136
gptejms said:
Why 'different fields'--you can have a superposition of two different field configurations,where the field is the same.

Yes, of course I meant two different configurations of the same field (let us say, the electron field).

What do you mean here?It's not the classical field,but the quantum field that obeys microscopic causality(commutation relations of the field).

No, the classical field satisfies microscopic causality (because of the fact that the differential equation (KG or Dirac) that it obeys is Lorentz-invariant). From this, and the quantization procedure the quantum operator associated with the field value at the point (x,t), satisfies commutation relations which correspond to microscopic causality, in the Heisenberg picture. Translated in the Schroedinger picture, this means that the UNITARY evolution of the wavefunction satisfies microscopic causality. However, collapse, which is not a unitary evolution, does NOT. So you're going to have a hard time to model this collapse by a unitary evolution, and that's what I'm trying to tell you since the beginning of this thread.
The above dichotomy is simply because QFT is just another quantum theory, just like non-relativistic QM is. And the "immediate collapse" is not something that has to do anything with the specific model of the dynamics we're trying to quantize (non-relativisitic point particles in Euclidean space, or classical fields obeying relativistic equations such as Dirac or KG). It is part of the general quantum theory. Even string theory suffers from it. So it doesn't matter what conditions you impose upon the dynamics (such as Lorentz invariance, or microscopic causality, or non-interaction, or whatever you can think of).
Collapse happens in Hilbert space, not in any real 4-dim manifold on which the quantum dynamics is modeled.

cheers,
Patrick.
 
  • #137
gptejms said:
Let me add here the following to what I already wrote in a part of post #132 as a response to the above.You are writing your wavefunction as a superposition of a 'field bump' at x1,t1 and a 'field bump' at x2,t1(i.e. two possible field configurations).Now your assertion is that when I make a measurement at x1,t1 and the bump appears there,the bump at x2,t1 is insatantaneously wiped off.

My assertion is that if at all you have to use your wavefunction for field superpositions,you can't use equal times at x1 and x2.At x2,the time must at least be (x2-x1)/c + t1 (in consistence with microscopic causality).

No, this is not the case. The superposition (in a fixed reference frame) is at equal times.
Let us call f1(x,y,z) = N1 exp(-(x^2 + y^2 - 81) ), the field config 1 at t = t1 (bump around circle in xy plane of radius 9)
and f2(x,y,z) = N2 exp(-(z^2 - 141)), the field config 2 at t=t1. (bump around z = 12)

Both field configurations correspond to orthogonal kets in Hilbert space, which we denote by |1> and |2> respectively.

I can now have the quantum field, at t=t1, in the state a|1>+ b|2>

If I measure now at t=t1, that the particle was around z=12, then suddenly the state of the quantum field collapses into |2>.

It doesn't even matter what is the time evolution (the dynamics) of the quantum field, and what conditions (such as microcausality) it satisfies, because this only relates to how the state |1> will evolve from t1 to t2, and how the state |2> will evolve from t1 to t2 (and as such, we know also how the above state a|1> +b|2> will evolve).
For instance, it might be that the evolved |1> will not be a single classical field configuration anymore at t2, but can be now a superposition of classical field configurations (this is highly probable, if the field configurations are "spiked" and we cannot apply the classical evolution equations: QFT effects are then visible). BUT ALL THIS HAS NOTHING TO DO with the projection postulate, which acts upon the quantum state at a given time.

Let me repeat that the Schrodinger equation is an equation for a quantum field not for wavefunction.Wavefunction in the new picture(if I may say so) has a lesser role to play--you can't use it for superpositions in the way one did for the wavefunction of a particle.

Well, I'm sorry but you have it completely backwards here (at least in standard QM). The Schroedinger equation - which is just as valid in QFT as in ordinary QM - is the evolution equation of the wavefunction (= the quantum state of the system) in Hilbert space. However, except for the single 1-particle non-relativistic QM case, the Schroedinger equation IS NOT an equation of a field over the spacetime manifold.

cheers,
Patrick.
 
  • #138
vanesch said:
No, the classical field satisfies microscopic causality (because of the fact that the differential equation (KG or Dirac) that it obeys is Lorentz-invariant). From this, and the quantization procedure the quantum operator associated with the field value at the point (x,t), satisfies commutation relations which correspond to microscopic causality, in the Heisenberg picture. Translated in the Schroedinger picture, this means that the UNITARY evolution of the wavefunction satisfies microscopic causality. However, collapse, which is not a unitary evolution, does NOT. So you're going to have a hard time to model this collapse by a unitary evolution, and that's what I'm trying to tell you since the beginning of this thread.

Ok,so in QFT,in the Schrodinger picture the field has the same status as the spatial coordinates of a wavefunction in NR quantum mechanics.The field is classical,and you may superpose different field configurations.
 
  • #139
vanesch said:
Well, I'm sorry but you have it completely backwards here (at least in standard QM). The Schroedinger equation - which is just as valid in QFT as in ordinary QM - is the evolution equation of the wavefunction (= the quantum state of the system) in Hilbert space. However, except for the single 1-particle non-relativistic QM case, the Schroedinger equation IS NOT an equation of a field over the spacetime manifold.

Look at it this way--you have the K.G. or the Dirac (quantum)field equations to begin with.Now you go on reducing the velocity till v/c <<1.How does the quantum field now change into a wavefunction?At what point does the transition occur?I am asserting that it remains a quantum field--the wavefunction even in the NR limit is a functional of the field.
 
  • #140
gptejms said:
Look at it this way--you have the K.G. or the Dirac (quantum)field equations to begin with.Now you go on reducing the velocity till v/c <<1.How does the quantum field now change into a wavefunction?At what point does the transition occur?I am asserting that it remains a quantum field--the wavefunction even in the NR limit is a functional of the field.

It's a trick :-) In fact, it turns out that the wave function in the QFT Hilbert space (which can be identified with the Fock space), when reduced to the 1-particle subspace of Fock space, satisfies the same equation as the classical field. But you can already see the difference, for instance in the case of a real classical field: the field is supposed to be real, and the resulting NR Schroedinger equation gives you complex 1-particle wave functions! Another way to see it is: how does your field reduce to the wave function of TWO PARTICLES ? The field is function of (x,t) and the wave function is function of (x1,x2,t).

There is just a "mathematical coincidence" of the wave function equation (Schroedinger equation) in the 1-particle case and the classical field equation to have the same form. Of course there are reasons why this is so, but it is not because the equations are the same, in this particular case, that the *objects* (classical field over 4-manifold vs. wavefunction in hilbert space) are the same.
But people who get confused are in good company: it took until the 50ies for people to realize the difference.
 

Similar threads

  • Quantum Physics
4
Replies
124
Views
3K
  • Quantum Physics
Replies
2
Views
839
Replies
4
Views
2K
Replies
15
Views
1K
  • Quantum Physics
Replies
4
Views
890
Replies
15
Views
987
Replies
1
Views
783
  • Quantum Physics
Replies
33
Views
2K
Replies
41
Views
2K
  • Quantum Physics
Replies
10
Views
1K
Back
Top